point load

Solution to Problem 687 | Beam Deflection by Method of Superposition

Problem 687
Determine the midspan deflection of the beam shown in Fig. P-687 if E = 10 GPa and I = 20 × 106 mm4.
 

Solution to Problem 686 | Beam Deflection by Method of Superposition

Problem 686
Determine the value of EIδ under each concentrated load in Fig. P-686.
 

Solution to Problem 685 | Beam Deflection by Method of Superposition

Problem 685
Determine the midspan value of EIδ for the beam loaded as shown in Fig. P-685. Use the method of superposition.
 

Solution to Problem 680 | Midspan Deflection

Problem 680
Determine the midspan value of EIδ for the beam loaded as shown in Fig. P-680.
 

Solution to Problem 674 | Midspan Deflection

Problem 674
Find the deflection midway between the supports for the overhanging beam shown in Fig. P-674.
 

Overhang beam with point load at the free end

 

Solution to Problem 673 | Midspan Deflection

Problem 673
For the beam shown in Fig. P-673, show that the midspan deflection is δ = (Pb/48EI) (3L2 - 4b2).
 

Simple beam with concentrated load

 

Solution to Problem 668 | Deflections in Simply Supported Beams

Problem 668
For the beam shown in Fig. P-668, compute the value of P that will cause the tangent to the elastic curve over support R2 to be horizontal. What will then be the value of EIδ under the 100-lb load?
 

Overhang beam with point load between supports and at the free end

 

Solution to Problem 667 | Deflections in Simply Supported Beams

Problem 667
Determine the value of EIδ at the right end of the overhanging beam shown in Fig. P-667. Is the deflection up or down?
 

Overhang beam with triangular and point loads

 

Solution to Problem 664 | Deflections in Simply Supported Beams

Problem 664
The middle half of the beam shown in Fig. P-664 has a moment of inertia 1.5 times that of the rest of the beam. Find the midspan deflection. (Hint: Convert the M diagram into an M/EI diagram.)
 

Simple beam with different moment of inertia over the span

 

Solution to Problem 661 | Deflections in Simply Supported Beams

Problem 661
Compute the midspan deflection of the symmetrically loaded beam shown in Fig. P-661. Check your answer by letting a = L/2 and comparing with the answer to Problem 609.
 

Symmetrically Placed Point Loads over a Simple Beam

 

Pages

Subscribe to RSS - point load